Difference between revisions of "Aufgaben:Exercise 2.6Z: Signal-to-Noise Ratio"

From LNTwww
m (Text replacement - "Category:Aufgaben zu Modulationsverfahren" to "Category:Modulation Methods: Exercises")
 
(13 intermediate revisions by 3 users not shown)
Line 3: Line 3:
 
}}
 
}}
  
[[File:P_ID1017__Mod_Z_2_6.png|right|frame|Verschiedene Leistungsdichtespektren]]
+
[[File:P_ID1017__Mod_Z_2_6.png|right|frame|Spectra and power density spectra]]
Wir gehen in dieser Aufgabe von folgenden Voraussetzungen aus:
+
In the following exercise,  we assume:
*ein cosinusförmiges Quellensignal:
+
*a cosine source signal:
 
:$$ q(t) = 4 \,{\rm V} \cdot \cos(2 \pi \cdot 5\,{\rm kHz} \cdot t )\hspace{0.05cm},$$
 
:$$ q(t) = 4 \,{\rm V} \cdot \cos(2 \pi \cdot 5\,{\rm kHz} \cdot t )\hspace{0.05cm},$$
* ZSB–AM durch Multiplikation mit
+
* DSB–AM by multiplication with
 
:$$z(t) = 1 \cdot \cos(2 \pi \cdot 20\,{\rm kHz} \cdot t )\hspace{0.05cm},$$
 
:$$z(t) = 1 \cdot \cos(2 \pi \cdot 20\,{\rm kHz} \cdot t )\hspace{0.05cm},$$
* eine frequenzunabhängige Dämpfung auf dem Kanal entsprechend  $α_{\rm K} = 10^{–4}$,
+
* a frequency-independent attenuation on the channel corresponding to  $α_{\rm K} = 10^{–4}$,
* additives weißes Eingangsrauschen mit Rauschleistungsdichte  $N_0 = 4 · 10^{–19} \ \rm W/Hz$,
+
* additive white input noise with power density  $N_0 = 4 · 10^{–19} \ \rm W/Hz$,
* phasen– und frequenzsynchrone Demodulation durch Multiplikation mit gleichem  $z(t)$  wie beim Sender,
+
* phase-synchronous and frequency-synchronous demodulation by multiplication the same  $z(t)$  as at the transmitter,
* ein rechteckförmiger Tiefpass beim Synchrondemodulator mit Grenzfrequenz  $f_{\rm E} = 5 \ \rm kHz$.
+
* a rectangular low-pass at the synchronous demodulator with cutoff frequency  $f_{\rm E} = 5 \ \rm kHz$.
  
  
In der Grafik sind diese Vorgaben im Spektralbereich dargestellt.  Ausdrücklich soll erwähnt werden, dass sich das Leistungsdichtespektrum  ${\it Φ}_z(f)$  der Cosinusschwingung  $z(t)$  ebenso wie das Amplitudenspektrum  $Z(f)$  aus zwei Diraclinien bei  $±f_{\rm T}$  zusammensetzt, aber mit dem Gewicht  $A^2/4$  anstelle von  $A/2$.  Die Amplitude ist bei dieser Aufgabe stets  $A=1$  zu setzen.
+
In the graph,  these specifications are shown in the spectral domain.  It should be explicitly mentioned that the power-spectral density  ${\it Φ}_z(f)$  of the cosine oscillation  $z(t)$  is composed of two Dirac delta lines at  $±f_{\rm T}$,  as in the amplitude spectrum   $Z(f)$,  but with weight  $A^2/4$  instead of  $A/2$.  The amplitude should always be set to  $A=1$  in this exercise.
  
Das Sinkensignal  $v(t)$  setzt sich aus dem Nutzanteil  $α · q(t)$  und dem Rauschanteil  $ε(t)$  zusammen.  Somit gilt allgemein für das zu bestimmende Signal–zu–Rausch–Leistungsverhältnis:
+
The sink signal  $v(t)$  is composed of the useful component  $α · q(t)$  and the noise component  $ε(t)$ .  Thus, the general rule for the signal-to-noise power ratio to be determined is:
 
:$$ \rho_{v } = \frac{\alpha^2 \cdot P_q}{P_\varepsilon}\hspace{0.05cm}.$$
 
:$$ \rho_{v } = \frac{\alpha^2 \cdot P_q}{P_\varepsilon}\hspace{0.05cm}.$$
Dieses wichtige Qualitätskriterium wird häufig mit  '''SNR'''  (englisch:  ''Signal–to–Noise–Ratio'') abgekürzt.
+
This important quality criterion is often abbreviated to  $\rm SNR$  ("signal–to–noise power ratio").
  
  
  
  
 
+
Hints:  
 
+
*This exercise belongs to the chapter  [[Modulation_Methods/Synchronous_Demodulation|Synchronous Demodulation]].
''Hinweise:''
+
*Particular reference is made to the pages  [[Modulation_Methods/Synchronous_Demodulation#Calculating_noise_power|Calculating noise power]]  and  [[Modulation_Methods/Synchronous_Demodulation#Relationship_between_the_powers_from_source_signal_and_transmitted signal|Relationship between the powers from source and_transmitted signal]].
*Die Aufgabe gehört zum  Kapitel  [[Modulation_Methods/Synchrondemodulation|Synchrondemodulation]].
+
*Please note that the variables  $α$  and  $α_{\rm K}$  need not be the same.  
*Bezug genommen wird insbesondere auf die Seiten   [[Modulation_Methods/Synchrondemodulation#Berechnung_der_Rauschleistung|Berechnung der Rauschleistung]]  sowie  [[Modulation_Methods/Synchrondemodulation#Zusammenhang_zwischen_.7F.27.22.60UNIQ-MathJax165-QINU.60.22.27.7F_und_.7F.27.22.60UNIQ-MathJax166-QINU.60.22.27.7F|Zusammenhang zwischen $P_q$  und  $P_{\rm S}$]].
+
*All powers refer to a resistance of  $R = 50 \ \rm Ω$  with the exception of subtask   '''(1)'''.
*Beachten Sie bitte auch, dass die Größen  $α$  und  $α_{\rm K}$  nicht unbedingt gleich sein müssen.  
+
*For DSB-AM without carrier,  $P_q$  also represents the transmit power  $P_{\rm S}$.
*Alle Leistungen mit Ausnahme der Teilaufgabe  '''(1)'''  beziehen sich auf den Widerstand  $R = 50 \ \rm Ω$.
 
*$P_q$  gibt bei „ZSB–AM ohne Träger” gleichzeitig die Sendeleistung  $P_{\rm S}$  an.
 
 
   
 
   
  
  
===Fragebogen===
+
===Questions===
  
 
<quiz display=simple>
 
<quiz display=simple>
  
{Berechnen Sie die Sendeleistung bezogen auf den Einheitswiderstand &nbsp;$R = 1 \ \rm Ω$.
+
{Calculate the transmit power with respect to the unit resistance &nbsp;$R = 1 \ \rm Ω$.
 
|type="{}"}
 
|type="{}"}
 
$P_q \ = \ $ { 8 3% } $\ \rm V^2$
 
$P_q \ = \ $ { 8 3% } $\ \rm V^2$
  
  
{Wie groß ist die Leistung &nbsp;$P_q$&nbsp; in „Watt” für den Widerstand &nbsp;$R = 50  \ \rm  Ω$?
+
{What is the power &nbsp;$P_q$&nbsp; in watts for the resistor &nbsp;$R = 50  \ \rm  Ω$?
 
|type="{}"}
 
|type="{}"}
 
$P_q \ = \ $ { 0.16 3% } $\ \rm W$
 
$P_q \ = \ $ { 0.16 3% } $\ \rm W$
  
  
{Welcher Dämpfungsfaktor &nbsp;$α$&nbsp; ergibt sich für das Gesamtsystem?
+
{Which attenuation factor &nbsp;$α$&nbsp; results for the whole system?
 
|type="{}"}
 
|type="{}"}
 
$α  \ = \ $ { 0.5 3% } $\ \cdot 10^{-4}$  
 
$α  \ = \ $ { 0.5 3% } $\ \cdot 10^{-4}$  
  
  
{Berechnen Sie die Leistungsdichte der Rauschkomponente &nbsp;$ε(t)$&nbsp; am Ausgang.&nbsp; Wie groß ist der Wert bei &nbsp;$f = 0$?&nbsp; Es gelte &nbsp;$H_{\rm E}(f = 0) = 1$.
+
{Calculate the power density of the noise component &nbsp;$ε(t)$&nbsp; at the output.&nbsp; What is the value when &nbsp;$f = 0$?&nbsp; Let &nbsp;$H_{\rm E}(f = 0) = 1$.
 
|type="{}"}
 
|type="{}"}
 
${\it Φ}_ε(f = 0) \ = \ $ { 4 3% } $\ \cdot 10^{-19} \ \rm  W/Hz$
 
${\it Φ}_ε(f = 0) \ = \ $ { 4 3% } $\ \cdot 10^{-19} \ \rm  W/Hz$
  
{Wie groß ist die Rauschleistung im Sinkensignal?
+
{What is the noise power of the sink signal?
 
|type="{}"}
 
|type="{}"}
 
$P_ε \ = \ $ { 4 3% } $\ \cdot 10^{-15} \ \rm  W$
 
$P_ε \ = \ $ { 4 3% } $\ \cdot 10^{-15} \ \rm  W$
  
  
{Wie groß ist das Signal–zu–Rausch–Leistungsverhältnis (SNR) an der Sinke?&nbsp; Welcher dB–Wert ergibt sich daraus?
+
{What is the signal-to-noise power ratio (SNR) at the sink? What is the resulting dB value?
 
|type="{}"}
 
|type="{}"}
 
$ρ_v \ = \ $ { 100000 3% }  
 
$ρ_v \ = \ $ { 100000 3% }  
Line 69: Line 67:
 
</quiz>
 
</quiz>
  
===Musterlösung===
+
===Solution===
 
{{ML-Kopf}}
 
{{ML-Kopf}}
'''(1)'''&nbsp; Das Leistungsdichtespektrum eines Cosinussignals mit der Amplitude&nbsp; $A$&nbsp; besteht aus zwei Diraclinien, jeweils mit Gewicht&nbsp; $A^2/4$.  
+
'''(1)'''&nbsp; The power-spectral density of a cosine signal with amplitude &nbsp; $A$&nbsp; consists of two Dirac delta lines,&nbsp; each with weight &nbsp; $A^2/4$.
*Die Leistung ergibt sich aus dem Integral über das LDS und ist somit gleich der Summe der beiden Diracgewichte.&nbsp; Mit&nbsp; $A = 4 \ \rm V$&nbsp; erhält man somit für die Leistung des Quellensignals:
+
*The power is obtained from the integral over the PDS and is thus equal to the sum of the two Dirac weights.&nbsp;
 +
*Thus,&nbsp; when &nbsp; $A = 4 \ \rm V$,&nbsp; we obtain the power of the source signal:
 
:$$ P_q = \frac{A^2}{2} \hspace{0.15cm}\underline {= 8\,{\rm V^2}} \hspace{0.05cm}.$$
 
:$$ P_q = \frac{A^2}{2} \hspace{0.15cm}\underline {= 8\,{\rm V^2}} \hspace{0.05cm}.$$
*Beim Modulationsverfahren „ZSB-AM ohne Träger” ist dies gleichzeitig die auf den Einheitswiderstand&nbsp; $1\ \rm Ω$&nbsp; bezogene Sendeleistung $P_{\rm S}$.
+
*For the modulation method&nbsp; "DSB-AM without a carrier",&nbsp; this is also the transmit power $P_{\rm S}$ in reference to the unit resistance&nbsp; $1\ \rm Ω$.
  
  
  
'''(2)'''&nbsp; Nach den elementaren Gesetzen der Elektrotechnik gilt:
+
'''(2)'''&nbsp; According to the elementary laws of Electrical Engineering:
 
:$$P_q = \frac{8\,{\rm V^2}}{50\,{\Omega}} \hspace{0.15cm}\underline {= 0.16\,{\rm W}} \hspace{0.05cm}.$$
 
:$$P_q = \frac{8\,{\rm V^2}}{50\,{\Omega}} \hspace{0.15cm}\underline {= 0.16\,{\rm W}} \hspace{0.05cm}.$$
  
  
  
'''(3)'''&nbsp; Im Theorieteil wird gezeigt, dass bei idealen Voraussetzungen&nbsp; $v(t) = q(t)$&nbsp; gilt.&nbsp; Zu berücksichtigen ist allerdings:
+
'''(3)'''&nbsp; In the theory section,&nbsp; it is shown that&nbsp; $v(t) = q(t)$&nbsp; holds under ideal conditions.&nbsp; However,&nbsp; the following should be taken into account:
*Aus der Grafik erkennt man, dass&nbsp; $Z_{\rm E}(f) = Z(f)$&nbsp; gilt.&nbsp; Damit hat das empfängerseitige Trägersignal&nbsp; $z_{\rm E}(t)$&nbsp; wie&nbsp; $z(t)$&nbsp; die Amplitude&nbsp; $1$.
+
*From the graph,&nbsp; it can be seen that&nbsp; $Z_{\rm E}(f) = Z(f)$&nbsp; holds.&nbsp; Thus,&nbsp; the receiver-side carrier signal &nbsp; $z_{\rm E}(t)$&nbsp; has like &nbsp; $z(t)$&nbsp; the amplitude &nbsp; $1$.
*Im Idealfall müsste aber das empfängerseitige Trägersignal&nbsp; $z_{\rm E}(t)$&nbsp; die Amplitude&nbsp;  $2$&nbsp; besitzen.
+
*Ideally,&nbsp; however,&nbsp; the receiver-side carrier signal &nbsp; $z_{\rm E}(t)$&nbsp; should have amplitude &nbsp;  $2$.
*Deshalb gilt gilt hier $υ(t) = q(t)/2$.  
+
*Therefore,&nbsp; $υ(t) = q(t)/2$&nbsp; applies here.  
*Berücksichtigt man weiter die Kanaldämpfung&nbsp; $α_{\rm K} = 10^{–4}$, so erhält man das Endergebnis: &nbsp; $α\hspace{0.15cm}\underline { = 0.5 · 10^{–4}}.$
+
*If we further consider the channel attenuation &nbsp; $α_{\rm K} = 10^{–4}$,&nbsp; we obtain the final result:&nbsp; $α\hspace{0.15cm}\underline { = 0.5 · 10^{–4}}.$
  
  
  
'''(4)'''&nbsp; Das Leistungsdichtespektrum des Produktes&nbsp; $n(t) · z(t)$&nbsp; ergibt sich aus der Faltung der beiden Leistungsdichtespektren von&nbsp; $n(t)$&nbsp; und&nbsp; $z(t)$:
+
'''(4)'''&nbsp; The power-spectral density of the product &nbsp; $n(t) · z(t)$&nbsp; is obtained by convolving the two power density spectra of &nbsp; $n(t)$&nbsp; and&nbsp; $z(t)$:
 
:$$ {\it \Phi}_\varepsilon \hspace{0.01cm} '(f) = {\it \Phi}_n (f) \star {\it \Phi}_{z }(f)= \frac{N_0}{2} \star \left[\delta(f - f_{\rm T}) + \delta(f + f_{\rm T}) \right]= N_0 \hspace{0.05cm}.$$
 
:$$ {\it \Phi}_\varepsilon \hspace{0.01cm} '(f) = {\it \Phi}_n (f) \star {\it \Phi}_{z }(f)= \frac{N_0}{2} \star \left[\delta(f - f_{\rm T}) + \delta(f + f_{\rm T}) \right]= N_0 \hspace{0.05cm}.$$
*Für das Leistungsdichtespektrum des Signals&nbsp; $ε(t)$&nbsp; nach dem Tiefpass erhält man eine Rechteckform mit dem gleichen Wert bei&nbsp; $f = 0$:
+
*For the power-spectral density of the signal &nbsp; $ε(t)$&nbsp; after the low-pass filter,&nbsp; we obtain a rectangular shape with the same value at &nbsp; $f = 0$:
 
:$${\it \Phi}_\varepsilon (f) = {\it \Phi}_\varepsilon \hspace{0.01cm} '(f) \cdot |H_{\rm E}(f)|^2 \hspace{0.3cm}\Rightarrow \hspace{0.3cm} {\it \Phi}_\varepsilon (f=0)= N_0\hspace{0.15cm}\underline {= 4 \cdot 10^{-19}\,{\rm W/Hz}} \hspace{0.05cm}.$$
 
:$${\it \Phi}_\varepsilon (f) = {\it \Phi}_\varepsilon \hspace{0.01cm} '(f) \cdot |H_{\rm E}(f)|^2 \hspace{0.3cm}\Rightarrow \hspace{0.3cm} {\it \Phi}_\varepsilon (f=0)= N_0\hspace{0.15cm}\underline {= 4 \cdot 10^{-19}\,{\rm W/Hz}} \hspace{0.05cm}.$$
  
  
  
'''(5)'''&nbsp; Die Rauschleistung ist das Integral über die Rauschleistungsdichte:
+
'''(5)'''&nbsp; The noise power is the integral over the noise power density:
 
:$$ P_{\varepsilon} = \int_{-f_{\rm E}}^{ + f_{\rm E}} {{\it \Phi}_\varepsilon (f)}\hspace{0.1cm}{\rm d}f = N_0 \cdot 2 f_{\rm E} = 4 \cdot 10^{-19}\,\frac{ \rm W}{\rm  Hz} \cdot 10^{4}\,{\rm Hz} \hspace{0.15cm}\underline {= 4 \cdot 10^{-15}\,{\rm W}}\hspace{0.05cm}.$$
 
:$$ P_{\varepsilon} = \int_{-f_{\rm E}}^{ + f_{\rm E}} {{\it \Phi}_\varepsilon (f)}\hspace{0.1cm}{\rm d}f = N_0 \cdot 2 f_{\rm E} = 4 \cdot 10^{-19}\,\frac{ \rm W}{\rm  Hz} \cdot 10^{4}\,{\rm Hz} \hspace{0.15cm}\underline {= 4 \cdot 10^{-15}\,{\rm W}}\hspace{0.05cm}.$$
  
  
  
'''(6)'''&nbsp; Aus den Ergebnissen der Teilaufgaben&nbsp; '''(2)''',&nbsp; '''(3)'''&nbsp; und&nbsp; '''(5)'''&nbsp; folgt:
+
'''(6)'''&nbsp; From the results of subtasks&nbsp; '''(2)''',&nbsp; '''(3)'''&nbsp; and&nbsp; '''(5)'''&nbsp; it follows that:
 
:$$\rho_{v } = \frac{\alpha^2 \cdot P_q}{P_\varepsilon} = \frac{(0.5 \cdot 10^{-4})^2 \cdot 0.16\,{\rm W}}{4 \cdot 10^{-15}\,{\rm W}} \hspace{0.15cm}\underline {= 100000} \hspace{0.3cm}\Rightarrow \hspace{0.3cm}10 \cdot {\rm lg }\hspace{0.1cm}\rho_{v } \hspace{0.15cm}\underline {= 50\,{\rm dB}}\hspace{0.05cm}.$$
 
:$$\rho_{v } = \frac{\alpha^2 \cdot P_q}{P_\varepsilon} = \frac{(0.5 \cdot 10^{-4})^2 \cdot 0.16\,{\rm W}}{4 \cdot 10^{-15}\,{\rm W}} \hspace{0.15cm}\underline {= 100000} \hspace{0.3cm}\Rightarrow \hspace{0.3cm}10 \cdot {\rm lg }\hspace{0.1cm}\rho_{v } \hspace{0.15cm}\underline {= 50\,{\rm dB}}\hspace{0.05cm}.$$
  
Line 110: Line 109:
  
  
[[Category:Modulation Methods: Exercises|^2.2 Synchrondemodulation^]]
+
[[Category:Modulation Methods: Exercises|^2.2 Synchronous Demodulation^]]

Latest revision as of 16:19, 18 January 2023

Spectra and power density spectra

In the following exercise,  we assume:

  • a cosine source signal:
$$ q(t) = 4 \,{\rm V} \cdot \cos(2 \pi \cdot 5\,{\rm kHz} \cdot t )\hspace{0.05cm},$$
  • DSB–AM by multiplication with
$$z(t) = 1 \cdot \cos(2 \pi \cdot 20\,{\rm kHz} \cdot t )\hspace{0.05cm},$$
  • a frequency-independent attenuation on the channel corresponding to  $α_{\rm K} = 10^{–4}$,
  • additive white input noise with power density  $N_0 = 4 · 10^{–19} \ \rm W/Hz$,
  • phase-synchronous and frequency-synchronous demodulation by multiplication the same  $z(t)$  as at the transmitter,
  • a rectangular low-pass at the synchronous demodulator with cutoff frequency  $f_{\rm E} = 5 \ \rm kHz$.


In the graph,  these specifications are shown in the spectral domain.  It should be explicitly mentioned that the power-spectral density  ${\it Φ}_z(f)$  of the cosine oscillation  $z(t)$  is composed of two Dirac delta lines at  $±f_{\rm T}$,  as in the amplitude spectrum   $Z(f)$,  but with weight  $A^2/4$  instead of  $A/2$.  The amplitude should always be set to  $A=1$  in this exercise.

The sink signal  $v(t)$  is composed of the useful component  $α · q(t)$  and the noise component  $ε(t)$ .  Thus, the general rule for the signal-to-noise power ratio to be determined is:

$$ \rho_{v } = \frac{\alpha^2 \cdot P_q}{P_\varepsilon}\hspace{0.05cm}.$$

This important quality criterion is often abbreviated to  $\rm SNR$  ("signal–to–noise power ratio").



Hints:


Questions

1

Calculate the transmit power with respect to the unit resistance  $R = 1 \ \rm Ω$.

$P_q \ = \ $

$\ \rm V^2$

2

What is the power  $P_q$  in watts for the resistor  $R = 50 \ \rm Ω$?

$P_q \ = \ $

$\ \rm W$

3

Which attenuation factor  $α$  results for the whole system?

$α \ = \ $

$\ \cdot 10^{-4}$

4

Calculate the power density of the noise component  $ε(t)$  at the output.  What is the value when  $f = 0$?  Let  $H_{\rm E}(f = 0) = 1$.

${\it Φ}_ε(f = 0) \ = \ $

$\ \cdot 10^{-19} \ \rm W/Hz$

5

What is the noise power of the sink signal?

$P_ε \ = \ $

$\ \cdot 10^{-15} \ \rm W$

6

What is the signal-to-noise power ratio (SNR) at the sink? What is the resulting dB value?

$ρ_v \ = \ $

$10 · \lg ρ_v \ = \ $

$\ \rm dB$


Solution

(1)  The power-spectral density of a cosine signal with amplitude   $A$  consists of two Dirac delta lines,  each with weight   $A^2/4$.

  • The power is obtained from the integral over the PDS and is thus equal to the sum of the two Dirac weights. 
  • Thus,  when   $A = 4 \ \rm V$,  we obtain the power of the source signal:
$$ P_q = \frac{A^2}{2} \hspace{0.15cm}\underline {= 8\,{\rm V^2}} \hspace{0.05cm}.$$
  • For the modulation method  "DSB-AM without a carrier",  this is also the transmit power $P_{\rm S}$ in reference to the unit resistance  $1\ \rm Ω$.


(2)  According to the elementary laws of Electrical Engineering:

$$P_q = \frac{8\,{\rm V^2}}{50\,{\Omega}} \hspace{0.15cm}\underline {= 0.16\,{\rm W}} \hspace{0.05cm}.$$


(3)  In the theory section,  it is shown that  $v(t) = q(t)$  holds under ideal conditions.  However,  the following should be taken into account:

  • From the graph,  it can be seen that  $Z_{\rm E}(f) = Z(f)$  holds.  Thus,  the receiver-side carrier signal   $z_{\rm E}(t)$  has like   $z(t)$  the amplitude   $1$.
  • Ideally,  however,  the receiver-side carrier signal   $z_{\rm E}(t)$  should have amplitude   $2$.
  • Therefore,  $υ(t) = q(t)/2$  applies here.
  • If we further consider the channel attenuation   $α_{\rm K} = 10^{–4}$,  we obtain the final result:  $α\hspace{0.15cm}\underline { = 0.5 · 10^{–4}}.$


(4)  The power-spectral density of the product   $n(t) · z(t)$  is obtained by convolving the two power density spectra of   $n(t)$  and  $z(t)$:

$$ {\it \Phi}_\varepsilon \hspace{0.01cm} '(f) = {\it \Phi}_n (f) \star {\it \Phi}_{z }(f)= \frac{N_0}{2} \star \left[\delta(f - f_{\rm T}) + \delta(f + f_{\rm T}) \right]= N_0 \hspace{0.05cm}.$$
  • For the power-spectral density of the signal   $ε(t)$  after the low-pass filter,  we obtain a rectangular shape with the same value at   $f = 0$:
$${\it \Phi}_\varepsilon (f) = {\it \Phi}_\varepsilon \hspace{0.01cm} '(f) \cdot |H_{\rm E}(f)|^2 \hspace{0.3cm}\Rightarrow \hspace{0.3cm} {\it \Phi}_\varepsilon (f=0)= N_0\hspace{0.15cm}\underline {= 4 \cdot 10^{-19}\,{\rm W/Hz}} \hspace{0.05cm}.$$


(5)  The noise power is the integral over the noise power density:

$$ P_{\varepsilon} = \int_{-f_{\rm E}}^{ + f_{\rm E}} {{\it \Phi}_\varepsilon (f)}\hspace{0.1cm}{\rm d}f = N_0 \cdot 2 f_{\rm E} = 4 \cdot 10^{-19}\,\frac{ \rm W}{\rm Hz} \cdot 10^{4}\,{\rm Hz} \hspace{0.15cm}\underline {= 4 \cdot 10^{-15}\,{\rm W}}\hspace{0.05cm}.$$


(6)  From the results of subtasks  (2)(3)  and  (5)  it follows that:

$$\rho_{v } = \frac{\alpha^2 \cdot P_q}{P_\varepsilon} = \frac{(0.5 \cdot 10^{-4})^2 \cdot 0.16\,{\rm W}}{4 \cdot 10^{-15}\,{\rm W}} \hspace{0.15cm}\underline {= 100000} \hspace{0.3cm}\Rightarrow \hspace{0.3cm}10 \cdot {\rm lg }\hspace{0.1cm}\rho_{v } \hspace{0.15cm}\underline {= 50\,{\rm dB}}\hspace{0.05cm}.$$